Menu Close

Question-155604




Question Number 155604 by SANOGO last updated on 02/Oct/21
Answered by Kamel last updated on 02/Oct/21
  ∀x≥0  x−(x^3 /6)≤sin(x)≤x     ∴   Σ_(k=1) ^n (1/(n+k))−(1/6)Σ_(k=1) ^n (1/((n+k)^3 ))≤Σ_(k=1) ^n sin((1/(n+k)))≤Σ_(k=1) ^n (1/(n+k))     ∴ lim_(n→+∞)   (1/n)Σ_(k=1) ^n (1/(1+(k/n)))−(1/6)lim_(n→+∞) (1/n^2 ) (1/n)Σ_(k=1) ^n (1/((1+(k/n))^3 ))≤lim_(n→+∞) Σ_(k=1) ^n sin((1/(n+k)))≤lim_(n→+∞) (1/n)Σ_(k=1) ^n (1/(1+(k/n)))  So:  ∫_0 ^1 (dx/(1+x))−lim_(n→+∞) (1/n^2 )∫_0 ^1 (dx/((1+x)^3 ))≤lim_(n→+∞) Σ_(k=1) ^n sin((1/(n+k)))≤∫_0 ^1 (dx/(1+x))     Then:  lim_(n→+∞) Σ_(k=1) ^n sin((1/(n+k)))=Ln(2)
$$ \\ $$$$\forall{x}\geqslant\mathrm{0}\:\:{x}−\frac{{x}^{\mathrm{3}} }{\mathrm{6}}\leqslant{sin}\left({x}\right)\leqslant{x} \\ $$$$\:\:\:\therefore\:\:\:\underset{{k}=\mathrm{1}} {\overset{{n}} {\sum}}\frac{\mathrm{1}}{{n}+{k}}−\frac{\mathrm{1}}{\mathrm{6}}\underset{{k}=\mathrm{1}} {\overset{{n}} {\sum}}\frac{\mathrm{1}}{\left({n}+{k}\right)^{\mathrm{3}} }\leqslant\underset{{k}=\mathrm{1}} {\overset{{n}} {\sum}}{sin}\left(\frac{\mathrm{1}}{{n}+{k}}\right)\leqslant\underset{{k}=\mathrm{1}} {\overset{{n}} {\sum}}\frac{\mathrm{1}}{{n}+{k}} \\ $$$$\:\:\:\therefore\:\underset{{n}\rightarrow+\infty} {{lim}}\:\:\frac{\mathrm{1}}{{n}}\underset{{k}=\mathrm{1}} {\overset{{n}} {\sum}}\frac{\mathrm{1}}{\mathrm{1}+\frac{{k}}{{n}}}−\frac{\mathrm{1}}{\mathrm{6}}\underset{{n}\rightarrow+\infty} {{lim}}\frac{\mathrm{1}}{{n}^{\mathrm{2}} }\:\frac{\mathrm{1}}{{n}}\underset{{k}=\mathrm{1}} {\overset{{n}} {\sum}}\frac{\mathrm{1}}{\left(\mathrm{1}+\frac{{k}}{{n}}\right)^{\mathrm{3}} }\leqslant\underset{{n}\rightarrow+\infty} {{lim}}\underset{{k}=\mathrm{1}} {\overset{{n}} {\sum}}{sin}\left(\frac{\mathrm{1}}{{n}+{k}}\right)\leqslant\underset{{n}\rightarrow+\infty} {{lim}}\frac{\mathrm{1}}{{n}}\underset{{k}=\mathrm{1}} {\overset{{n}} {\sum}}\frac{\mathrm{1}}{\mathrm{1}+\frac{{k}}{{n}}} \\ $$$${So}:\:\:\int_{\mathrm{0}} ^{\mathrm{1}} \frac{{dx}}{\mathrm{1}+{x}}−\underset{{n}\rightarrow+\infty} {{lim}}\frac{\mathrm{1}}{{n}^{\mathrm{2}} }\int_{\mathrm{0}} ^{\mathrm{1}} \frac{{dx}}{\left(\mathrm{1}+{x}\right)^{\mathrm{3}} }\leqslant\underset{{n}\rightarrow+\infty} {{lim}}\underset{{k}=\mathrm{1}} {\overset{{n}} {\sum}}{sin}\left(\frac{\mathrm{1}}{{n}+{k}}\right)\leqslant\int_{\mathrm{0}} ^{\mathrm{1}} \frac{{dx}}{\mathrm{1}+{x}} \\ $$$$\:\:\:{Then}:\:\:\underset{{n}\rightarrow+\infty} {{lim}}\underset{{k}=\mathrm{1}} {\overset{{n}} {\sum}}{sin}\left(\frac{\mathrm{1}}{{n}+{k}}\right)={Ln}\left(\mathrm{2}\right) \\ $$

Leave a Reply

Your email address will not be published. Required fields are marked *